You are on page 1of 200

1 A 24-year -old man with a history of drug abuse has nausea , rhinorrhea , hypertension, and tachycardia The most

likely cause is withdrawal from which of the following agents?

O A) Alcohol
ft B) Amphetamine
O C) Cocaine
G D) Heroin
ft E) Nicotine

D. Page 617 discusses psychoactive drug intoxication and withdrawal.

Opioids are depressants, so withdrawal causes the opposite and leads to "flu-like" symptoms.
Rhinorrhea is the main give-away here for opioid withdrawal.
38 A 45-year-old man comes to the physician because of progressively worsening, constant pain in his left thigh over the past
3 months. He is a long-distance runner An x-ray of the femur shows thickening of the diaphysis and disruption of the cortex
with focal areas of increased calcification A glistening mass containing several cysts is surgically excised. A
photomicrograph of tissue from the mass is shown. Which of the following is the most likely diagnosis?
Q A) Atypical stress fracture
O B) Chondrosarcoma
O C) Enchondroma
© D) Ewing sarcoma
O E) Giant cell carcinoma
O F) Multilobulated bone cyst
O G) Osteosarcoma

B. Page 510 talks about bone tumors.

Chondrosarcoma = mainly affects the axial skeleton than the appendicular skeleton. Malignant. t s
Enchondroma = mainly affects the hands and feet; are cyst like. Benign. „ «
#
The histology shown has pleomorphic cells characteristic of a malignant tumor.
I
i
*
«
*
Malignant tumors include osteosarcoma, chondrosarcoma, and ewing sarcoma.

Osteosarcoma occurs in the metaphysis of long bones and the knee and has pleomorphic
osteoblasts that cause a Codman triangle. This is usually secondary to Paget disease of
km *' 4
i
the bone, retinoblastoma, or Li-Fraumeni syndrome.

Ewing occurs in the diaphysis of long bones and the pelvis and has anaplastic small blue cells
of mesencyhmal origin. It is associated with the EWS-FLI1 t(11;22) translocation.
- I
«
42 A 50- year -old man is found dead in bed at home . His family states that he had chest pain for 3 weeks prior to his death At i
autopsy, examination shows severe atherosclerosis of all three major coronary arteries, with myocardial softening and
mottling involving the anterolateral wall of the left ventricle A photomicrograph of a section from this area is shown. Which t
* *

of the following is the most likely mechanism of death? v > *


4
** r
© A) Arrhythmia i
V
i *A <4
• r
*
© B) Cardiogenic shock
O
O
C)
D)
Myocardial rupture
Papillary muscle dysfunction
•i
%

t :
1
n- k

E) Reperfusion injury

-.
•i •V
A. Page 323 discusses evolution of an MI.
» *

*•
\ - 1 z **
4

t 4
1

Histology shows coagulative necrosis (preserved architecture of myocardial fibers) with


4
*
>
C .* Vi*« i

neutrophil infiltration which hinted that the MI was within 24 hours. Most likely cause of I
i
? «\ L;
*
y
'
l -I
death within first 24 hours of MI is arrhythmia. « I

i
The stem also describes myocardial rupture, which would be visible on gross appearance. i * »* S ‘i
* V

. *
However, the question asks the mechanism of death and the most common cause of i *« '. I
*•
l
death is initially an arrhythmia. i
/
4

«
I I «
v c
I
7
.v 1 M
• « i

*
34 A 70- year -old woman is found to have persistent fever despite intravenous broad-spectrum antibiotic therapy
3 days after undergoing operative excision of the anterior pelvic organs for recurrent cervical carcinoma Her
temperature is 38.5°C (101 3°F) Physical examination shows the presence of a central venous catheter that was
placed on the day of the operation There is also a well- healing abdominal wound without erythema and with two
drains in place Blood cultures and cultures of the tip of the central catheter on a sheep blood agar plate grow the
organism shown in the photograph A Gram stain of one of the colonies shows 4-pm , elliptical, purple , budding
organisms Which of the following is the most likely causal organism?

O A) Candida albicans
B) Cryptococcus neoformans
© C) Escherichia coli
D) Sporothnx schenckii
O E) Staphylococcus aureus

A. Page 155 discusses Candida albicans.

Candida is a part of the normal flora of skin, could cause contamination of a central venous catheter.
The question states that the organism is purple, budding, and that did not respond to broad spectrum
antibiotics (aka they didn't use fluconazole or amphotericin B). The yeast form of Candida is gram +.
Lastly, they showed it plated on blood agar and there was no hemolysis which eliminates staph aureus.

Cryptococcus usually involves meningitis in immunocompromised pts.


E. coli is gram negative.
Sporothrix is usually transmitted by a thorn on a rose or someone with a history of gardening.
43. A 66-year -old woman comes to the physician because of fever, chills, and left lower abdominal pain for 1 day She has had Increasingly severe constipation over the past 5 years
A barium study of the lower gastrointestinal tract shows three separate , poorly delimited regions of narrowing of the lumen of the distal sigmoid A photograph of one of the lesions
in the resected large intestine is shown Which of the following is the most likely diagnosis?

© A) Angiodysplasia
B) Diverticulitis B. Page 416 discusses diverticula.
C) Granulomatous colitis
O D) Ischemic colitis The clinical presentation is that of lower left quadrant abdominal pain, fever, and chills
9 E) Ulcerative colitis should alone make you think it was an inflammatory process.
Also the question says there are 3 separate poorly delimited regions of narrow lumen.
As far as ulcerative colitis is concerned, there are no skip lesions, it is continuous wherever it is.
Angiodysplasia would have hematochezia due to dilation of vessels of the right colon, aortic stenosis or vWF disease.
This coupled with the history of constipation makes diverticulitis the best answer choice.

The picture confused many people but it is really just an unnecessary distractor.
This patient's severe constipation for the past 5 years is what caused the diverticulosis to start.

Generally, in the elderly, LLQ pain = Diverticulitis and RLQ pain = angiodysplasia.
8 A 51-year-old woman comes to the physician because of a 6-month history of burning abdominal pain that occurs 1 to 2 hours after eating. She sweats profusely and has light-
headedness when she stands Her blood pressure is 105/70 mm Hg while standing . Physical examination shows epigastric tenderness A CT scan of the abdomen shows a 2-cm
mass on the proximal duodenum. Gastrin released by the tumor cells stimulates which of the following labeled cells in the photomicrograph shown to release hydrogen ions?

Area labelled B is the parietal cell.

Images -- http://www.siumed.edu/~dking2/erg/GI178b.htm
Left to Right
A, B, C, and E.

5 . A 65 -year -old man dies in a motor vehicle collision. A photograph of a lung as seen at autopsy is shown. Which of the
following is the most likely diagnosis?
A) Abscess
O B) Hamartoma
Q C) Infarct
D) Malignant lymphoma
O E) Mesothelioma
F) Squamous cell carcinoma

F.

Squamous cell is centrally located and has cavitation, which you can see in the pic.
Similar to this one: https://webpath.med.utah.edu/LUNGHTML/LUNG068.html

Macroscopically, squamous cell carcinoma tends to be off-white in color,


arising from, and extending into a bronchus.
10 A 22-year-old man comes to the physician because of a 2-month history of foul-smelling, watery diarrhea with significant flatulence; he
also has had a 4 5-kg (10-lb) weight loss during this period . He recently returned from a trip to rural Indonesia, where he did not always
,

have access to clean water He appears thin Physical examination shows a soft , mildly distended abdomen with active bowel sounds A
photomicrograph of a stool specimen is shown. The most appropriate pharmacotherapy has which of the following mechanisms of action?
© A) Enhancement of cell membrane permeability to chloride ions
B) Formation of destructive free radicals
Q C) Inhibition of DNA polymerase
© D) Inhibition of protein synthesis
E) Prevention of microtubule assembly

# * ' •»
r

I *
.T\
/» % v

10pm

B. Page 159 discusses Giardia lamblia. Page 201 discusses metronidazole.

DOC for giardia = metronidazole. MOA of metronidazole = formation of toxic free radical metabolites in the bacterial cell wall that damage
DNA making it bactericidal and antiprotozoal. Metro treats = GET GAP = giardia, entamoeba, trichomonas, Gardnerella, anaerobes (below
diaphragm), and H. pylori (as an alternative to amoxicillin in PCN allergy). Adverse effects = disulfiram-like reaction, HA, and metallic taste.
38 A 25-year-old nulligravid woman comes to the physician because she and her husband have not been able to conceive a child for 3 years. Previous evaluation of the husband
showed no abnormalities in spermatogenesis. Menarche was at the age of 13 years Menses occur at irregular 60- to 90-day intervals. She is 157 cm (5 ft 2 in) tall and weighs 68
kg (150 lb): BMI is 28 kg /m 2. Physical examination shows hirsutism. Ultrasonography shows ovaries with numerous cysts beneath the capsule. A photograph of the uterus and the
ovaries from a woman with a similar condition is shown. This patient is at greatest risk for developing which of the following conditions?
© A) Adenomyosis
© B) Dermoid cyst D. Page 703 discusses PCOS.
O C) Ectopic pregnancy
O D) Endometrial hyperplasia PCOS is associated with abnormal production of sex steroids, including dysfunction of estrogen production
O E) Endometriosis and progesterone. Chronically elevated levels of estrogen can cause endometrial hyperplasia, possibly
© F) Hydatidiform mole
leading to endometrial cancer.
© G) Leiomyomata uteri
© H) Meigs syndrome
O I) Placenta previa
10 A photograph is shown of a myelin- stained cross section of a spinal cord obtained at autopsy from a 50-year -old
woman She sustained a spinal cord injury in a motor vehicle collision 15 years prior to her death Which of the
following symptoms would most likely have been present in this patient as a result of the damage indicated in the
photograph?
A) Atrophy of the arm muscles
B) Atrophy of the thigh muscles
C) Hyperreflexia of the quadriceps deep tendon reflex
D) Hyperreflexia of the triceps deep tendon reflex
E) Loss of sensation to vibration over both feet
F) Loss of sensation to vibration over both hands

E. Page 550 discusses spinal cord tracts.

This patient's spinal cord is stained for myelin but the pink portion is damaged and has a lack of myelin.
That pink portion is the gracilis portion of the dorsal columns, which are sensory fibers for the legs.

The lateral corticospinal tract or lower motor neuron lesions would be related to atrophy of arm or thigh muscles.

Hyperreflexia is seen in upper motor neuron lesions.


37 A 62-year-old man who is a farmer comes to the physician because of an 8-year history of the lesions indicated by
the arrows shown in the photograph Although there is no itching, he occasionally picks at the lesions , which causes
some bleeding Physical examination shows numerous similar rough plaques on the scalp , face , ears , and forearms
Which of the following is the most likely diagnosis?
© A) Actinic keratosis A. Page 526 discusses Actinic Keratosis.
© B) Basal cell carcinoma Students got confused from the "bleeding" in the question
© C) Hand dermatitis and felt it might be psoriasis, but the lesions are not
© D) Psoriasis psoriatic at all.
O E) Seborrheic keratosis
© F) Squamous cell carcinoma
The farmer picks carrots.
O G) Tinea corporis
H) Verruca vulgaris
Farmer = Sun exposure and SqCC
Carrot = Keratosis
14 A 68-year -old woman has had a fever and shortness of breath for the past week X-rays of the chest show a right lower lobe infiltrate.
Leukocyte count is 38 ,000/mm 3 A peripheral blood smear is shown. Which of the following best describes her peripheral blood smear?
© A) Acute lymphoid leukemia
O B) Acute myeloid leukemia
C) Chronic lymphocytic leukemia
O D) Reactive granulocytosis
© E) Reactive lymphocytosis

D. Page 456 discusses left shift, and page 465 discusses Leukemias.
Acute lymphoid leukemia (ALL): Look at these MONSTER lymphocytes in the 1st image above. High N/C ratio.
Acute myeloid leukemia (AML): Watch out for the faggot cells (a faggot is a bundle of sticks).
The sticks are crystals made of MPO. Look at set of images below.
Chronic lymphocytic leukemia (CLL): Smudge cells are characteristic.
Can transform into diffuse large B-cell lymphoma (Richter transformation).
C. Page 564 discusses Neurodegenerative disorders.

Huntington's disease. Autosomal dominant.

Anticipation: She has a similar disorder as her father but died earlier.
Remember "HUNT 4 an animal, put it in a CAGe". Huntingtin gene found on Chromosome 4.
Huntingtin causes histone deactylation, silencing genes necessary for neuronal survival.
CAG is the trinucleotide repeat: Caudate loses ACh and Gaba.
Chorea, caudate nucleus, Ataxia, Gloomy (depression)

5 A 43-year-old woman dies after a 10-year illness characterized by irregular jerking movements of the extremities and a disjointed gait She had profound dementia during the last
2 years of her life Her father had a similar disorder and died at the age of 65 years At autopsy, examination of this patient's brain is most likely to show severe atrophy of which of
the following labeled structures in the MRI of the normal brain?
29 A 43-year-old man comes to the physician for a follow -up examination. Ten years ago, he underwent cardiac valve replacement. Chest x-rays are shown Based on these findings,
which of the following valves was most likely replaced in this patient?
O A) Aortic
O B) Mitral A
G C) Pulmonic
© D) Tricuspid
21 A 37- year -old man is brought to the emergency department 45 minutes after sustaining injuries in a motor vehicle collision He has severe , sharp , right -sided chest pain His
respirations are 30/min and labored Physical examination shows numerous areas of trauma to the right side of the head and thorax Breath sounds are decreased on the right ,
and there is hyperresonance in the right thorax Physical examination shows decreased tactile fremitus, and adventitious sounds are absent X -rays confirm multiple rib fractures
A CT scan of the chest is shown The most likely afferent pathway of this patient’s chest pain is carried in which of the following nerves?
® A) Greater thoracic splanchnic B.
9 B) Intercostal
O C) Phrenic With this question, I think they want you to recognize that the patient isn't having chest pain related to the heart.
9 D) Pulmonary parasympathetic
They emphasize several rib fractures and a pneumothorax but don't indicate any heart damage
O E ) Vagus
(lack of adventitious sounds = no pulmonary edema indicated or a lung issue related to heart problem).

The pericarditis is what's innervated by the phrenic nerve.


Seeing as how his heart is fine, the fractured ribs are probably what are causing him pain
via the intercostal nerves.
37 A 14-year-old girl is brought to the physician by her mother because of a 2-month history of hair loss on her head The mother says that her
daughter has been trying to imitate the hairstyles of several pop music stars by using various ties, rubber bands , and curlers , but she is not
sure whether any chemical treatments have been used The mother reports that the patient has been sad since her grandmother died
unexpectedly 3 months ago. Vital signs are normal . Physical examination shows decreased hair density over several irregular patches
temporally and frontally, but no denuded areas The underlying skin is normal The remaining hair shafts in these thinned areas are of
varying lengths There is no frontotemporal recession A photograph of the affected area is shown Which of the following is the most likely
explanation for this patient's hair loss?
O A) Alopecia areata
© B) Androgenetic alopecia
O C) Telogen effluvium
Q D) Tinea capitis
Q E) Trichotillomania

E. Page 609 discusses trichotillomania.

The clues for trichotillomania were the death


of her grandmother suddenly, since
trichotillomania is often stress-induced.

The hair was in different growth stages in the


patchy areas, which makes sense if she’s
plucking them out at different times.

Telogen effluvium is most common in


middle-aged women, so she doesn’t fit
the profile (but you’re right about it
being caused by stress). However, this and
alopecia would have denuded areas.
47 A 27-year-old primigravid woman at 34 weeks' gestation is admitted to the hospital because of nausea and vomiting, and abdominal pain for 12 hours Her pregnancy has been
otherwise unremarkable. Prior laboratory studies, including a complete blood count and serologic tests for HIV and hepatitis, have shown no abnormalities. Her blood pressure now
is 164/102 mm Hg Abdominal examination shows right upper quadrant tenderness. Laboratory studies show:
Hemoglobin 7 4 g/dL
Hematocrit 24%
Platelet count 72,000/mm 3
Serum
Creatinine 1 2 mg/dL
Total bilirubin 2 3 mg/dL
AST 112 U/L
ALT 126 U/L

Examination of a peripheral blood smear is most likely to show a predominance of which of the following in this patient?
A) Acanthocytes D. Page 701 discusses hypertension in pregnancy.
B) Heinz bodies
C) Howell-Jolly bodies HELLP syndrome: Hemolysis Elevated Liver enzymes Low Platelets.
D) Schistocytes
E) Spherocytes A manifestation of severe preeclampsia. Blood smear shows schistocytes.
F) Target cells Can lead to DIC and hepatic subcapsular hematomas -->rupture -->severe hypotension.

Deliver the baby immediately.


26

© A) Adhesions
© B) Embolism
O C) Intussusception
© D) Strangulation
© E) Volvulus

D. Page 401 discusses hernias.

The inguinal part of the question points you to a a femoral hernia.

Femoral hernia: uncommon type of inguinal hernia, in which intra-abdominal contents


(in this case the small bowel) herniate into the femoral canal through the femoral ring.

The image: strangulation: ischemic necrosis of contents within the hernia sac as blood
flow is compromised due to incarceration (irreducible femoral hernia due to trapped
hernia contents in hernia sac).

So, in short: femoral hernia can lead to a complication called incarceration which can lead to strangulation.
Tyrosine

i Tyrosinase

Dopaquinone
Low cysteine environ High cysteine environ

Dopachrome CD-quinones

I TYRP2
Intrinsic chemical
5 ,6-Dihydroxyindole- reactivities
2-carboxylic acid

I TYRP1

Eumelanin (black ) Pheomelanin (red )


21 A 5-year-old boy is brought to the physician for a well-child examination. He recently immigrated to the USA from Nigeria with his parents. He has no history of major medical
illness He is at the 40th percentile for height and weight. His vital signs are within normal limits Physical examination shows red- bronze skin , red hair, and continual horizontal
nystagmus. Laboratory studies show a complete blood count that is within the reference range and normal leukocyte and platelet morphology. Genetic testing shows compound
heterozygosity of the TYRP 1 gene in the figure shown, with the presence of both a frame-shift (368delA ) and a nonsense (S166X ) mutation These genetic changes most likely
caused which of the following in this patient?
O A) Absence of tyrosinase in the affected cells
B) Decreased concentrations of 5 ,6-dihydroxyindole-2-carboxylic acid
0 C) Decreased reactive oxygen species in pigmented cells
0 D) Increased photoprotection of the prevalent melanin form
© E) Lack of choroidal pigment deposition in the macula

E.

Proccess of elimination for this one. A & B you can eliminate immediately just from looking at the biochemical pathway chart.
The other two required knowledge that eumelanin is more protective than pheomelanin (this is why redheads burn more easily).
Because pheomelanin is less protective, there would be more ROS from sunlight.
28. A 19-year-old woman is brought to the emergency department 30 minutes after falling on her outstretched right hand during a gymnastics
competition Physical examination shows tenderness of the anatomical snuff-box. An x-ray of the right wrist is shown. The most likely
diagnosis is fracture of which of the following carpal bones?
© A) Capitate
© B) Hamate
Q C ) Lunate
O D) Scaphoid
O E) Triquetrum

D. Page 481 discusses wrist bones.

Snuffbox -- Scaphoid.
The anatomic snuffbox is formed by the tendons of the extensor pollicis brevis,
the abductor pollicis longus, and the extensor pollicis longus (all 3 are radial nerve posterior forearm muscles).

The floor is formed by the scaphoid bone, and it is here that one can palpate for a possible fractured scaphoid.
left The most likely cause of her
woman develops the acute onset of spastic left hemiparesis and weakness of the lower two thirds of the face on the
22 A 66-year - old right-handed the MRI of the brain shown?
condition is a lesion at which of the following labeled areas on

B. Page 573 discusses motor neuron signs.

First of all, B and E are the only options on the right


side of the brain. They labelled it, but in general
remember that these images have you looking at the
patient from their feet.

Spastic paralysis is an UMN lesion, which should


immediately make you think of an internal capsule stroke.

O A) O B) O C) O D) O E)
I OrO
1 2

II
1 2 3 4
65

III
1
••^
- 2 3

# Affected female
OD
6 7
4 5

Affected male
'totD'OO
8 9 10 11 12 13 14 15
6
16 17 18

O Unaffected female
Unaffected male
0 Affected male, fetal death

17 A 26- year -old woman (III-2) comes to the physician for counseling prior to conception Many members of her family had blistering of the skin in infancy and now have unusual
marbled pigmentation of the skin The underlying inherited condition is uniformly fatal to males in utero A pedigree is shown Which of the following is the most likely risk that this
patient's live- born offspring will be affected?
O A) 25% in females but near 0 in males
D. Page 59 discusses modes of inheritance.
© B) 25% in males but near 0 in females
© C) 25% regardless of the sex of the child The question asks about "live-born offspring" and affected males die in utero, so 0 males will
© D) 50% in females but near 0 in males be born affected. The disease is given from the mother to males and females relatively equally,
© E) 50% in males but near 0 in females so it must be X-linked dominant.
O F) 50% regardless of the sex of the child
t> G) Nearly 100% The disease in this question is incontinentia pigmenti, an XLD disorder that affects skin, hair,
O H) Not increased over the general population risk teeth, nails, and the CNS. https://en.wikipedia.org/wiki/Incontinentia_pigmenti
50. A 65-year-old woman undergoes surgical repair of an aneurysm of the right internal carotid artery in the cavernous sinus. Three days later, physical examination shows that the
right pupil is larger than the left pupil There is also weakness with movement of the eye A diagram of a coronal section through the middle cranial fossa, including the cavernous
sinus and associated structures , is shown. Which of the following labeled nerves is most likely damaged in this patient?

A. Page 587 discusses the cavernous sinus.

Cranial nerve 3.
The abducens n. is actually the nerve most likely to be damaged by an expanding internal carotid aneurysm in
the cavernous sinus but they give you specific CN3 function in this question.
C is the abducens nerve CN6 while the others on the wall are CN3, 4, 5.1 and 5.2 and they are rarely damaged.
Cavernous sinus syndrome presents with variable opthalmoplegia depending on the nerve involved, Horner syndrome, and more.
41 A 54- year-old man comes to the physician for a routine health maintenance examination His pulse is 80/min, respirations are 14/min, and blood pressure is 140/95 mm Hg
Physical examination shows no other abnormalities An angiotensin l-converting enzyme inhibitor is prescribed Which of the following labeled regions in the illustration of the
kidney shown will be most affected by the administration of this drug?

F. Page 632 discusses filtration.

ATII constricts the efferent arteriole.


ACE inhibitors block the ACE-mediated
conversion of ATI to ATII.

Remember ACE= Angi2 Constricts Efferent


and an Ace Inhibitor will stop that.
33. A study is conducted on patients with asthma to compare a new asthma treatment with a placebo After 12 weeks of therapy the mean (+ standard error of the mean ) symptom
scores were 2.8 + 0.1 in the treated group (n=103) as compared to 3 1 + 0 1 in the placebo group (n=100) The p-value is 0 035 The investigators of this study state that they
,

chose a sample size of about 100 subjects in each group to have an 80% power of detecting a mean difference of 0 4 at a significance level of 5%. When planning a study, which of
the following is the meaning of "80% power"?
C A) If 80% of the subjects improve with the drug: there is a 95% chance that a study of this size will find a
p-value > 0.05
O B) If 80% of the subjects improve with the drug, there is a 95% chance that a study of this size will find a
p-value < 0 05
> C) If the treatment decreases the asthma score by 0 4 in 80% of the subjects, there is a 95% chance that a study of this size will find a p-value > 0 05
Q D) If the treatment decreases the asthma score by 0.4 in 80% of the subjects , there is a 95% chance that a study of this size will find a p-value < 0 05
Q E) If the treatment really changes the mean asthma score by 0 4 , there is an 80% chance that a study of this size will find a p-value > 0.05
O F) If the treatment really changes the mean asthma score by 0.4 , there is an 80% chance that a study of this size will find a p-value < 0.05

F. Page 270 discusses incorrect results.

This question essentially asks you to define and apply power given the study they are conducting.
Power = 1 - beta and gives you the true positive rate and helps lower the probability of a type 2 error.

If power is 80%, then there is a 80% chance you will have a true positive result, and 20% chance for a false-negative.
There is an 80% chance that the null hypothesis will be rejected because a difference will actually exist.

So if the treatment detects a mean difference of 0.4 in asthma in the patients in the treatment group,
then that data falls in the 80% power range, and its significance is 95% (p<0.05 so it did not happen by chance).
12. A 52-year-old woman is admitted to the hospital because of shortness of breath due to malignant ascites and pleural effusion She has had ovarian cancer for 3 years. Treatment
has included several courses of aggressive chemotherapy including intraperitoneal chemotherapy and sclerotherapy for her recurrent pleural effusion During prior hospitalizations,
she has undergone debulking procedures She has been discouraged by the ineffectiveness of therapy, and she feels emotionally and physically exhausted Her primary care
gynecologist suggests a palliative care consultation, but her gynecologic oncologist says he is "absolutely opposed to giving up" and is recommending referral for inclusion in a
clinical trial. This disagreement between the two physicians is disconcerting to the patient Following thoracentesis and paracentesis , the patient calls her primary care internist to
help address this conflict between the two physicians. Which of the following is the most appropriate next step for the internist to address the patient's concerns?

C A) Contact a representative from the hospital risk management department


C B) Speak with the chair of the hospital ethics committee
O C) Speak with the two gynecologists to attempt to coordinate care
C D) Refer the patient for pastoral counseling
G E) Refer the patient for a psychiatric consultation to deal with her anxiety about the conflict

C.

It is just most evidently C. There is no doubt about it and the other choices just dont make any sense whatsoever.
12 A 65-year-old woman is being treated in the hospital for widely metastatic breast cancer unresponsive to chemotherapy She has never married and has no children or siblings.
Throughout her illness, another woman she had introduced as "my close friend," has been with her during hospitalizations and office visits The patient is moved to an inpatient
hospice after she decides she wants no further curative therapy She and her friend appear very distressed when they read a sign outside the hospice unit that states only family
members are permitted to stay after visiting hours Her friend says, "We can't bear to be apart It would be cruel to separate us now. " Which of the following responses by the
physician is most appropriate?

A) "Don't worry I'll call you right away if something happens to your friend Just leave your number with the nurses ”
B) "I'm sorry, but only family is allowed to stay with patients in the hospital You will have to leave after your 15 minutes are up "
C) "The two of you seem to have a very important relationship Of course you may stay together "
D) "Visiting hours are posted outside. You are welcome during any of those times I'm sorry, but an exception cannot be made in your case "
E) "Well, I guess we could bend the rules just this time, but if anyone asks, you're her sister. Does that sound OK?"

C.

Generally speaking, this question had fundamental issues for many students.
Dont be mean, so let them stay together. But then you break the rule of the hospital. And E clearly is not acceptable, but C does not
seem alright because B is the truth of the matter. Because of that, A should have been the most logical and acceptable answer choice.

Some others said that maybe they are significant others now because of how she was introduced as a "close friend."
In addition, the patient has absolutely no family, so this other woman will be the family in this case because the patient has deemed it so.
25 A 34-year-old woman comes to the physician because she has been feeling weepy and overwhelmed since delivering a healthy male newborn 6 weeks ago. She has had fatigue
and irritability during this period, and she has had no interest in engaging in activities she used to enjoy prior to the birth of the baby. She also has had difficulty sleeping because
she is always listening for sounds indicating that her baby is awake She says. "My husband doesn't help me at all with the baby. We argue all the time now " She then says, "I feel
guilty because I'm not enjoying my baby more I don't know if I will be a good mother or not " Physical examination shows no abnormalities. Which of the following statements by the
physician is most appropriate?

© A) "Has it come as a surprise to you how hard parenting is? Many people feel that way ”
O B) "I’m concerned about how bad you’ve been feeling lately. Have you had any thoughts about death or wanting to be dead?"
G C) "I’m sure that you'll feel better soon After all, look at your beautiful baby."
O D) "Most new mothers feel this way, and sometimes it helps to see a therapist. Would you like me to give you a referral?"
© E) "Your relationship with your husband sounds strained. I don't blame you for being angry with him."

B. Page 608 discusses peripartum mood disturbances.


Postpartum blues usually resolve within 10 days.
MDD with peripartum onset lasts more than 2 weeks. This is what the patient is exhibiting.
Postpartum psychosis involves thought of harming the baby or self and requires hospitalization and atypical antipsychotics.

The only good answer is the one that is gauging how far her peripartum disturbance may have gone.
A.

Answer A is the most amicable choice but many students felt there were some discrepancies. This is a bullshit/garbage question.

This question is two-fold. You must know the decision-making hierarchy and also consider autonomy.
Making "recommendations" for certain treatments is wrong, because it violates autonomy and informed consent.
Physicians should simply explain the RBA's of all treatment options, and allow the family to decide what is best based
on informed consent and "their perceptions of the PATIENT'S wishes". If there is no advanced directive, then the
decision should be appealed to a surrogate decision-maker.

Priority of surrogates is spouse-->adult children-->parents-->siblings.


Yes, the spouse would have higher priority, but the family should at least try to reach a consensus first before appealing to "only" the wife.
If a consensus, cannot be made, then the wife would have first say in the decision or things can be moved to the ethics committe.

Therefore, encouraging the family to decide according to the wishes of the PATIENT would be a better first step, especially
if the wishes of the wife is contrary to that of the patient.
B. Page 459 discusses platelet disorders.

The patient suffered from Immune Thrombocytopenia,


autoantibodies against the glycoproteins GP2B/3A.
B. Page 643 discusses acidosis and alkalosis.

Na-(Cl-+HCO3-)=anion gap = 140-(105+10) = 25 mEq/L

The normal range for anion gap is 8 to 16 mEq/L so this patient has an increased anion gap.

The pH = 7.25, and the normal range 7.35-7.45, so this patient has acidosis.

Normal bicarbonate is 22-28, and this patient has a bicarbonate of 10, so this must be metabolic acidosis.

Additionally PCO2 is normally 33-45 mm Hg, the question shows PCO2 of 23 with normal PO2, so there is respiratory compensation.
C. Page 6-3 discusses childhood and early onset disorders.

"Excitatory amino acids" refers to glutamate, while "Biogenic" apparently refers to tyrosine, the precursor AA to dopamine and norepi.
Amphetamines use the NE transporter (NET) to enter the presynaptic terminal, where they utilize the vesicular monoamine transporter
(VMAT) to enter neurosecretory vesicles. This displaces NE from the vesicles. Once NE reaches a concentration threshold within the
presynaptic terminal, the action of NET is reversed, and NE is expelled into the synaptic cleft.

This child most likely has ADHD and amphetamines are the treatment for most of these disorders.
A. Page 564 discusses neurodegenerative disorders and page 593 discusses the related drugs.

This patient's progressive memory loss over 9 months with no signs of depression and a normal physical exam
strongly suggest Alzheimer's disease. AChE inhibitors are used for treatment because Alzheimer's is associated
with low ACh. Also used are NMDA receptor antagonists like memantine.
A.

Applying PEEP increases alveolar pressure and alveolar volume.


The increased lung volume increases the surface area by reopening and stabilizing collapsed or unstable alveoli.
This splinting, or propping open, of the alveoli with positive pressure improves the ventilation-perfusion match,
reducing the shunt effect.
E.

Expanding the interstitial fluid volume exerts radial tension on the anchoring filaments attached to endothelial cells of initial lymphatics,
which increases luminal volume. This creates a small pressure difference that drives interstitial fluid into the lymphatic vessels.
In addition, increased luminal diameter reduces the resistance to lymph flow.

Interstitial pressure would increase with IV saline.


E. Page 369 discusses diagnosing parathyroid disease and page 656 discusses renal osteodystrophy.

This patient has secondary hyperPTHism due to chronic renal failure.


Her creatinine level is absurdly high, so her kidney basically is not working.
Her body is making PTH but her kidney is not responding so her body will make more PTH.
The excess PTH has lowered her calcium.
Her bone pain is due to renal osteodystrophy.
The patient will have more 25-hydroxycholecalciferol because the liver's 25-hydroxylase will be
functional but the kidney's 1a-hydroxylase will not work, but this is not the direct cause of her bone pain.
A. Page 148 discusses V cholerae.

Vibrio cholerae is a gram-negative, comma shaped bacteria that can cause watery diarrhea.
Cholera toxin functions by activating the Gs proteins --> increasing activity of adenylyl cyclase
--> increased cAMP --> increased Na+ and Cl- efflux --> diarrhea.
D. Page 273 discusses decision-making capacity.

Stem actually states, “On questioning, the patient does not know the date [time], the name of the hospital [place], or the name of her nurse
who had just introduced himself [person].” So, pt is disoriented to time and place (Choice A); that is definitely concerning -- as would be
depressed mood (Choice E) and the other choices -- but “inability to understand severity and prognosis” is the most concerning since that
is the very definition of capacity.

Inability to understand = lack of capacity. Being informed is the "i" of GIEMSA.


D. Page 70 discusses rickets.

Children with rickets have pathologic bow legs (genu varum), bead-like costochondral junctions (rachitic rosary), craniotabes (soft skull).
Rickets has defective mineralization of osteoid and cartilaginous growth plates.
C. Page 209 discusses HIV therapy.

DIDanosine causes PancreaDIDis.


Just a memorization question.
A.

Good simple video --> https://www.youtube.com/watch?v=4-DuvwoH2zQ

This one was tricky but I think you could’ve done this one without knowledge of NMDA receptors.
Stem told you that glutamate activates both non-NMDA and NMDA receptors but it activated only non-NMDA receptors in the early phase.
That means NMDA receptors activate after non-NMDA receptors.
That means something was delaying NMDA receptor activating and the only answer that made sense as the Mg inhibiting
NMDA at resting potential. Once the cell is depolarized by non-NMDA receptors, NMDA receptors can be activated.
D.

"Hungrybox" on Reddit said-


The 2 commandments of ethics questions:

Don't ever pick an answer where you sound like a dick.


Don't ever consult the ethics committee.
B. Page 528 discusses the arachidonic acid pathways.
If you block off COX enzymes with ibuprofen, there will be more arachidonic acid and a lot of it will be shunted to making more LTC4.
This is the basis of NERD, Nsaid Exacerbated Respiratory Disease.
E. Page 694 discusses Tanner stages of development and precocious puberty.

This patient is 8 and the stages usually start at 8-11 years so if the question stem had posed a younger patient, we would be cautious.
Being in the 50th percentile for height is not that significant because she can suddenly get taller or maybe she will just be short.
B. Page 238 discusses the Lineweaver Burk plot.
The Lineweaver Burk Plot is mainly interpreted by its X and Y intercepts.
The question states that giving B6 increases the activity to normal levels, which means the activity should be the same as the normal.
Hence, their Y intercept should be the same, so we are between choices B and C. A and D do not have the same Y intercept as normal.
Higher concentrations of B6 caused activity to become normal, and so Vmax will not be changing and we can cancel A and D.
A has a lower Vmax and D has a much higher Vmax.
The difference between B and C is in their Km. Moving to the left on the X axis makes the Km lower and tells you that affinity is higher
so you would not need more B6. But in our case affinity of the enzyme for B6 is really low, which is why we need a ton more B6.

In summary, we want the same Vmax and a higher Km.


We want the "normal" activity (same vmax as normal) and we need higher amounts of B6 for success so
affinity of the enzyme for B6 is probably very low.

Choice B has a (-1/Km) value closer to 0 which means Km is lower and affinity of the enzyme for its substrate is super low.
This makes sense as giving higher amounts of the "competitive" substrate B6 is helping.
B. Page 595 discusses opioid analgesics.

Morphine is a mu opioid full agonist - one adverse effect of opioids is mast cell degranulation that is IgE-independent.
Release of histamine is akin to an anaphylactic reaction --> pruritis, etc.
D. Page 169 discusses parvovirus.

Healthy people who get sick with parvovirus may have a subclinical illness with athralgias as the only symptom.
But sickle cell patients and patients with decreases hematopoiesis will develop[ bone marrow failure because the
virus affect the hematopoietic stem cells leading to aplastic crises and inability for the bone marrow to adequately
compensate hence the low reticulocyte count.

Parvovirus causes slapped cheeks in kids hydrops fetalis in fetus, aplastic crises in sickle cell and arthralgias in other patients.
D. Page 399 discusses biliary structures.

This is another name for the ampulla of Vater.


Choice E was just a bait answer choice.
E. Page 569 discusses neurocutaneous disorders.

The constellation of symptoms sounds like tuberous sclerosis.


Cardiac rhabdomyoma is a rare benign tumor that is frequently associated with tuberous sclerosis.
Tuberous sclerosis has subependymal giant cell astrocytomas and ungual fibromas.
C. Page 428 discusses hemochromatosis.

This patient has the bronze diabetes symptoms in this question.


C. Page 457 discusses heme synthesis and porphyrias.

This patient has issues with UV sensitivity so the enzyme you go for is Uroporphyrinogen decarboxylase.
The patient has porphyria cutanea tarda.
This is also associated with HepC so that might explain the high liver enzymes.
D.

Conceptual understanding really helps.


The esophagus is retroperitoneal and the jugular veins are far from that aspect of things.
So after that, the lung starts to make much more sense.
A. Page 376 discusses MEN Syndromes.

Recurrent kidney stones should include hyperparathyroidism on your differential.


Couple that with gastrinoma and you’re looking at MEN 1. Lipomas are also associated with MEN 1.
A.

Although it is unclear, people argue this could be Dengue or Zika.


Regardless the vector is the same for them all.
Dengue break bone fever and Zika arthralgias are all good give-aways for this stem.
B. Page 409 discusses esophageal cancer.

Squamous cell carcinoma occurs in the upper 2/3 of esophagus whereas adenocarcinoma
occurs in the distal 1/3. Since this was in the mid esophagus, its squamous cell carcinoma.
Key feature of squamous cell carcinoma is keratin pearls.
D. Page 66 discusses vitamin B1.

Wernicke-Korsakoff syndrome due to thiamine (B1) deficiency. Common in alcoholics.

The reason why they said "results of alcohol and drug screen are negative" is because she might be a chronic alcoholic.

Wernicke's triad:
Confusion
Paralysis of eye muscles (ophthalmoplegia - *presents here as nystagmus)
Ataxia

Korsakoff's psychosis:
Memory loss (anterograde and retrograde)
Making shit up (confabulation)
Personality change

She probably hasnt drank recent enough to have a positive screening but her B1 and mamillary bodies are already influenced.
B. Page 743 discuses asthma.

Many students chose D but this patient’s main symptom is cough only during exercise.
This is more indicative of exercised associated asthma.
You could see shortness of breath in MVP during exercise,
but choosing MVP leaves the cough unaccounted for.
A. Page 269 discusses measures of central tendency.

The mode is the number that is repeated the most number of times.
Group X has 32 people with a dBP of 70, while group Y has 20 people with a dBP of 80.
So the mode of X is 70 and the mode of Y is 80.

The median is the number in the center when all 100 people are listed in increasing order.
Listing everything out might take time, but it is evident that the median would probably be
around 70 for group X and around 80 for group Y. So you can choose A or list it all out.
C. Page 360 discusses adrenal insufficiency.

The stem is describing primary adrenal insufficiency, or Addison's.

ACTH is being over-produced to stimulate the adrenals to produce cortisol, but they can't respond,
either due to atrophy or destruction (TB, autoimmune: DR4, etc.)
The first 13 amino acids of ACTH can be cleaved to form α-MSH, which stimulates melanocytes, causing hyperpigmentation.
Cortisol helps with BP and his is low.

Patients also have low aldosterone.


Low Na and high K is a sign of hypoaldosteronism. Patients retain H and lose HCO3.
Losing HCO3 causes Cl- retention in the PCT. This all leads to metabolic acidosis.
Loss of cortisol causes anorexia, hypoglycemia, and a low BP as seen in this patient.
C. https://www.youtube.com/watch?v=Kps3VzbykFQ

"ITT analysis includes every subject who is randomized according to randomized treatment assignment.
It ignores noncompliance, protocol deviations, withdrawal, and anything that happens after randomization."
C. Page 108 discusses interferon gamma. Page 109 discusses interferons in general.

INFg is secreted by NK cells and T cells in response to antigen or IL12.


INFg activates macrophages to kill phagocytosed pathogens and inhibits Th2 differentiation.
It also activates NK cells to kill virus infected cells, but D is wrong, IL3 (stimulates bone marrow) is not involved nor is this significantly increased.
A. Page 365 discusses hypothyroidism.

This patient has Hashimoto. The question stem asks what would happen if her TSH is high.
Her TSH being high can only mean she has hypothyroidism and she is trying to correct it.
Hypothyroidism causes cretinism in the fetus.
B. Page 369 discusses Parathyroid disease.

Hyperparathyroidism causing bone lesions is via PTH inducing Osteoblasts to release IL1 and RANK -L expression
to bind to RANK on Osteoclasts and stimulating them ---> inc Bone Resorption.
H. Page 475 discusses Microtubule inhibitors.

"Taxes stabilize society."


Taxanes stabilize metaphase, so anaphase cannot occur.
Used for ovarian and breast cancers. Because ovaries and breasts run the society.
Sort of weird that theyre saying to use it for bladder cancer and throw students off.
C. Page 51 discsusses Osteogenesis imperfecta.

The primary structure is glycine-x-y, and with the tiny glycine being replaced by the larger alanine,
the secondary structure will be messed up.

A is wrong because the hydrogen bonds are between amino hydrogen and carboxyl oxygen atoms.
C. Page 140 discusses Listeria.

Couldnt be Strep agalactiae because it does not really make the mother sick.
Normal
7.35 < pH < 7.45
36 < pCO2 < 44
20 < HCO3 < 28

B. Page 634 discusses acidosis and alkalosis.

It has been 45 minutes since his wife found him, but it hasnt been too long so this is still pretty acute.
This patient's HCO3 is low and PCO2 is high, so that indicates this patient has acidemia.
Here, we see that the CO2 is high on the ABG.
This means that patient is hypoventilating since levels of CO2 are ventilation dependent, and also that patient has respiratory acidosis.
Also, bicarb is low, which implies that it's being "soaked up" by metabolic acidosis.

Also;
Henderson-Hasselbach Equation: pH = 6.1 + log(HCO3 / (0.03 * PaCO2))= 6.1+log6.6=6.92
So here, pH = 6.92 => Acute (uncompensated) primary respiratory acidosis, with metabolic acidosis.

On FA, in the graph, this falls in the pH of 6.9 and hco3 of 12, which is mixed acidosis.

Generally, first look at the pH, and that is not given. Then differentiate between respiratory and metabolic based off of pCO2 (normal 40).
This patient has a high pCO2, so this patient must have CO2 retention leading to respiratory acidosis. Now figure out if it is acute or chronic.
Kidneys compensate for the high H+ by resorbing HCO3, which requires 3-5 days.
However, in our question, HCO3 is high because of the concomitant metabolic acidosis
Very evidently D.
D. Page 522 discusses common skin disorders.

This patient has inflammation of her skin due to heat, spicy food, and other external stimuli.
This is not acne, there are no comedones.

Pityriasis rosea (page 526) has a patch that spreads into a Christmas tree distribution over the trunk.
It will be pink plaques that resolve in 2 months time.
E. Page 39 discusses Glitazones.

Remember that Glitazones work with PPARg and fibrates work with PPARa.
E. Page114 discusses blood transfusion reactions.
Pretty obvious.
C. Page 323 discusses MI.

Mixed venous oxygen tension is the amount of oxygen bound to hemoglobin as it returns
to the right side of the heart after traveling to the tissues.

Decreased oxygen delivery to the tissues due to decreased cardiac output leads to the
tissues extracting more oxygen from the passing RBCs than they normally would, which

Decreased cardiac output will cause peripheral vasoconstriction so SVR would be increased partly due to RAAS.
Pulmonary vascular resistance increases in efforts of getting more blood out of the lungs and into the body.
D. Page 76 discusses Glycolysis.

If BPG is elevated, it means the enzyme after is not working. BPG is made after PFK1 does its job.
Later pyruvate kinase makes pyruvate. If you have a lack of glycolytic enzymes, you get hemolytic anemia.
E.

Very straightforward question. You can see that because of diabetes, the baby is very big.
A. Page 451 discusses anemias and the iron panel.

This patient has a low TIBC because their total iron levels are okay.
The other choices dont even make sense and IDA is not a choice.
Patients with RA can have ACD.
B. Page 340 discusses lipid-lowering agents.

Statins and Fibrates both cause myopathy and the risk is increased when you take both concurrently.
D. Page 149 discusses Syphilis.

“The pathologic changes associated with syphilis are characterized by obliterative endarteritis that is found in all stages of the disease.”
https://www.ncbi.nlm.nih.gov/pmc/articles/PMC2811633/
A. Page 340 discusses lipid-lowering agents.

The most appropriate vitamin for this patient would be related to lipid-lowering agents, and that would be vitamin B3, Niacin.
Niacin reduces hepatic VLDL synthesis and causes results similar to statins by reducing LDL and increasing HDL.
Niacin also inhibits hormone-sensitive lipase, inhibiting lipolysis in adipose tissue.
A.

It is important to make the patient's environment accomodating.


Starting therapy would not help maintain her functional level and she does not yet need the therapy.
Assisted living is not needed yet for her either.
A. Page 94 discusses Familial dyslipidemias.

They described the "creamy layer in supernatant" as the "turbid" fasting plasma.
In Type 1 FD, Hyperchylomicronemia, there is a lack of Lipoprotein Lipase or its cofactor C2.
C. Page 617 discusses psychoactive drugs.

Toluene, the main component of volatile glues, lacquer thinners and aerosol paints is the chemical responsible for most clinical toxicity.
Inhalants cause an initial excitatory response through the release of epinephrine and activation of the dopamine system, followed by
central nervous system depression mediated by the use of GABA pathway. It manifests as a sense of euphoria, excitation, dizziness,
disinhibited behaviour and exhilaration similar to alcohol intoxication, thus resulting in psychological dependence.

Repeated inhalations by the user to prolong the intoxication will develop in headache, slurred speech, diplopia, gait abnormality,
delusions, visual hallucinations and disorientation.

Cocaine would not present in this manner. It would also have bleeding in the nose.
Ethanol or alcohol would be harder for a minor to access and its symptoms would be similar but also have smell of alcohol.
Meth is a stimulant.
PCP would make him aggressive.
D. Page 133 discusses endotoxin. Page 99 discusses TLR.

LPS found in outer membrane of gram ⊝ bacteria (both cocci and rods).
Composed of O antigen + core polysaccharide + lipid A (the toxic component).

Activates Macrophages and induces TNFalpha release -> Hypotension and fever.
D. Page 613 discusses sexual dysfunction.

Patient is having psychogenic ED - its all in his head, libido is normal and nocturnal erections are normal.

If his Testosterone level was low - Libido decreased.


If his nervous system (parasympathetic/sympathetic) was damaged - nocturnal erections decreased.
F. Page 190 discusses sexually transmitted infections.

The only tender/painful STIs are H ducreyi chancroids and HSV2 herpes.
Chancroids are eruptive, deep ulcers. This patient has vesicles and ulcers. HSV is vesicular.
C. Page 120 discusses immunosuppressants.

Cyclosporine and Tacrolimus both prevent IL2 transcription.


Both are nephrotoxic but cyclosporine causes hairyness, while tacrolimus causes diabetes.
Sirolimus prevents response to IL2 and also induces insulin resistance but is not nephrotoxic.

Thus, you can combine Cyclosporine and Sirolimus.


C. Page 559 discusses central poststroke pain syndrome.
B. Page 322 discusses aortic dissections.

Major risk factor for aortic dissection is hypertension, and in this case might be due to cocaine use,
which causes marked hypertension. Dissections cause a tear in the tunica intima -- blood can flow
backwards into the pericardium and cause tamponade. This manifests as crackles in the lung due
to poor left ventricular function (filling/diastolic problem due to compression).

Another clue is the "diminished pulses" only in the left arm, which means that the left subclavian artery
must be involved and an aortic dissection is the only plausible explanation here.

The diastolic murmur is aortic regurgitation due to dilation of the aortic root.
E. Page 231 discusses Li-Fraumeni, oncogenes and tumor suppressors.

Li-Fraumeni syndrome = SBLA (sarcoma, breast, leukemia, adrenal gland syndrome) and occurs because of an
autosomal dominant inherited mutation of p53.

APC: linked to FAP (colorectal cancer)


RET: linked to papillary thyroid cancer, MEN 2A, MEN 2B
RB1: retinoblastoma
D.

AV Fistulas re-rout blood from the arterial system to the venous system, by-passing the Arterioles = Increase PL ---> INCREASE VR. All in all =
Increase CO.

According to UWorld, the arterioles are a major source of resistance ... so bypassing the arterioles results in a decrease in Total Peripheral
Resistance ... causing an increase in the rate and volume of blood returning to the heart. I am pretty sure there is more to the physiology behind
this, but I hope this explained a little.

"Immediately following creation, arteriovenous fistula (AVF) is associated with an increase in cardiac output (CO), achieved predominantly
through a reduction in systemic vascular resistance, increased myocardial contractility, and an increase in stroke volume (SV) and heart rate.
Over the following week, circulating blood volume increases in conjunction with increases in atrial and brain natriuretic peptides. These
alterations are associated with early increases in left ventricular (LV) filling pressure with the potential for resultant impact on atrial and
ventricular chamber dimensions and function." (PMID: 25258554) There's also another study by Epstein from the 1950s looking at the effects of
AVF's effect on CO in men (PMID: 13052718). Apparently, the increase in resting CO is a big problem because it can lead to high-output cardiac
failure (LVH).
E. Page 723 discusses Lung anatomy.

Pleural space: Midcavicular line: 6-8th ribs Midaxillary line: 8-10th ribs Paravertebral line: 10-12nd ribs

So physician must insert a needle in 8-10th ribs in midaxillary line; but insertion below the 9th rib still has
a risk to damage abdominal organs such as liver. Upper border of 9th rib is fine.
A.

There are two essential fatty acids: linoleic = omega 6, and alpha-linoleic = omega 3.
C. Page 696 discusses diorders of sexual development.

This video discusses genetic differentiation really well -- https://www.youtube.com/watch?v=ciQjo7bj-uQ

1- SRY proteins stimulate development of testes.


2- This pt has testes => he must have the SRY gene on the Y chromosome.
3- MIF degrades the Mullerian duct, which would otherwise become the internal female genitalia.
4- This pt has internal female genitalia => didn't make enough MIF.

Since the testes are normal, eliminate choices A and B. Of the remaining choices, C is most common.
C. Page 653 discusses urinary incontinence.

This video discusses the micturition reflex very clearly -- https://www.youtube.com/watch?v=US0vNoxsW-k

This patient has overflow incontinence.

Stress incontinence; leakage with coughing. Exercise levator ani. Use midodrine (alpha 1 agonist) to contract urinary
sphincter and detrusor. Use duloxetine (SNRI) to increase noradrenergic and serotonergic activity in the sacral
spinal cord for more EUS activity. This is seen with EUS or pudendal nerve trauma, specially in women during delivery.

Urge incontinence; sudden urge to empty bladder.


Muscarinic M3 antagonists (tolterodine, oxybutynin,
solifenacin, dicyclomine) and beta 3 agonists like
mirabegron can relax detrusor muscles to stop
the detrusor overactivity. Loss of the hypogastric nerve
leads to loss of sympathetic innervation, so now you
keep needing to pee thanks to overpowering
parasympathetic innervation.

Overflow incontinence; incomplete bladder emptying


leads to a leak with overfilling. Muscarinic M2 agonists
like Bethanechol used to treat detrusor underactivity.
Use a -zosin alpha 1 antagonists if BPH related.
The pelvic nerve is parasympathetic, which is for rest
and digest; urination. Loss of parasympathetic nerve
leads to loss of bladder contraction. Bethanechol is
a parasympathomimietic drug. Sacral S1 demyelination
in MS patients or diabetic autonomic neuropathy
can cause this.
B. Page 595 discusses Baclofen.

Pt appears to have Muscle spasticity as a result of MS.


Spasticity is treated with a muscle relaxant like Baclofen which is a GABA agonist.
Bethanacol is a Cholinomimentic which can be used to treat Urinary dysfunction in ms.
Question asks what would help treat the spasticity.
A. Page 585 discusses ocular motility.

Many people cancelled out A and chose E because of the impaired upward gaze.
In the last sentence it asks you to assume an "entrapment", so it is actually the inferior rectus which
is the cause of the upward gaze palsy. The entrapped muscle is functionally trapped in it's shortened
position, thereby not allowing the orbit to gaze upward.
Also, these are the only muscles on the orbital floor.
E. Page discuses hypertensive emergency treatment, and page 501 discusses vascular smooth muscle contraction.

This patient evidently has hypertensive emergency (BP is 210 and he has end-organ damage).
The drugs that help are nitroprusside, clevidipine, fenoldopam, labetalol, nicardipine, and nitroprusside.
Nitrates increase cGMP. Think of viagra, phosphodiesterase inhibitors, and vasodilation.
The choices with a red line depict choices that do the opposite of most anti-hypertensives.
You need NO, you block calcium channels, and you stop calcium release from the sarcoplasm for HTN treatment.
And cAMP is more related to sympathetics and glucagon.
B.

Ubiquitin-mediated proteolysis is not reversibly affected by insulin.


The question asks for reversible ways that insulin affects it, and ubiquitination would lead to degradation via proteases, which is not reversible.
Nuclear/cytoplasmic shuttling makes sense because FOXO is a transcription factor that needs to leave the nucleus.
Also, remember that insulin works with the Mitogen-activated Protein (MAP) serine/threonine kinase pathway (pg 358).

https://www.youtube.com/watch?v=ewgLd9N3s-4
C. Page 747 discusses sarcoidosis.

This patient has all the signs of sarcoidosis.


There is increased 1a-hydroxylase mediated vitamin D activation in the noncaseating granulomas, leading to hypercalcemia.
B. Page 679 discusses reproductive anatomy.

The uterus is very internal, so internal iliac. Its very straightforward. The uterine artery is the branch.
The external iliac becomes the femoral artery after the inguinal ligament and that is unrelated to the pelvis.
The ovarian artery comes directly from the aorta, like the testicular arteries.
D. Page 510 discusses osteosarcoma.

It’s an osteosarcoma. Sarcoma = hematogenous mets. It’s in the legs so think how a DVT goes to lung. Same idea.
B. Page 121 discusses recombinant cytokines.

The patient has Neutropenic fever. GSCF will restore his neutrophils.
His RBC and platelets are low, but acceptable levels for someone undergoing chemotherapy.
Erythropoietin also seems like a plausible answer choice, but, since he has an infection and
is taking antibiotics, it is important that his neutrophils be working.
In addition, note that there are two answer choices for neutrophils, but you can strike out
D because IL8 (clean up at aisle 8) is just a neutrophil chemotactic factor.
C. Page 214 discusses apoptosis.

Orchiectomy = ↓ testosterone production = ↓ DHT => prostate cells undergo apoptosis.


With the drop in testosterone (and therefore DHT), some of the prostate cells will undergo apoptosis.
Apoptosis = DNA fragmentation (180 bp segments).
(This mechanism is similar to using 5α-reductase blockers to treat BPH.)

Apoptosis is characterized by DNA fragmentation (pyknosis, karyorrhexis, karyolysis).


Note that degradation in apoptosis is progressive.
From pyknosis -> karyorrhexis -> karyolysis. Aka condensation -> fragmentation -> complete dissolution.
A. Page 562 discusses headaches.
A. Page 749 discusses respiratory physiology, and page 752 discusses PHTN.

Hypoxic pulmonary vasoconstriction (HPV), also known as the Euler-Liljestrand mechanism, is a physiological phenomenon
in which small pulmonary arteries constrict in the presence of alveolar hypoxia (low oxygen levels).

While the maintenance of ventilation/perfusion ratio during regional obstruction of airflow is beneficial, HPV can be detrimental
during global alveolar hypoxia which occurs with exposure to high altitude, where HPV causes a significant increase in total pulmonary
vascular resistance, and pulmonary arterial pressure, potentially leading to pulmonary hypertension and pulmonary edema.

This is still acute. Eventually, this will be chronic and he will make more EPO and not be hypoxic anymore.
D. Page 556 discusses the progression of a stroke.

The stroke occurred 16+1 = 17 hours ago. Within the first 24 hours you see red neurons.
Then neutrophils, macrophages, astrocytes, and a scar.
C. Page 266 discusses precision vs accuracy.
D. Pages 737-738 discuss carbon dioxide transport.

The majority of carbon dioxide molecules are carried as part of the bicarbonate buffer system.
In this system, carbon dioxide diffuses into the RBCs.
Carbonic anhydrase (CA) within RBCs quickly converts the carbon dioxide into carbonic acid (H2CO3).
Carbonic acid is an unstable intermediate molecule that immediately dissociates into bicarbonate ions (HCO3-) and hydrogen (H+) ions.

The newly synthesized bicarbonate ion is transported out of the RBC into the plasma in exchange for a chloride ion (Cl−);
this is called the chloride shift. When the blood reaches the lungs, the bicarbonate ion is transported back into the RBC in exchange for
the chloride ion. The H+ ion dissociates from the hemoglobin and binds to the bicarbonate ion. This produces the carbonic acid intermediate,
which is converted back into carbon dioxide through the enzymatic action of CA.
The carbon dioxide produced is expelled through the lungs during exhalation.

This is all related to the Bohr-Haldane effect.


C. Page 396 discusses Portosystemic anastomoses.

Upon examination of the answer choices, only choices B and C have a portosystemic shunt.
Of the two, the splenic shunt seems more relevant to the esophageal bleeding this patient has.

This question is about the splenorenal shunt, which is not in FA.


https://my.clevelandclinic.org/health/treatments/4950-distal-splenorenal-shunt
C. Page 513 discusses Gout.

This is a case of acute gout. Monosodium urate crystals are taken up by neutrophils, leading to an acute inflammatory reaction.
T-cells aren't really involved in gout (more rheumatoid arthritis).
Think about how neutrophils got there; increased vascular permeability.
B. The vaccines are discussed on page 111.

Capsular polysaccharide vaccines are often conjugated to proteins to improve immunogenicity.


Flagellin is the only answer choice that's a protein.

This would be a subunit vaccine, which is used for HBV, HPV, aP, neisseria, strep pneumo, and Hib.
E.

Splenectomy predisposes patients to be infected by encapsulated organisms.


E coli and Strep pneumo are both encapsulated.

The patient just had a car accident and got a splenectomy due to the trauma.
He is now EQUALLY susceptible to pretty much both.

This question is not good in all honesty.


However, strep pneumo is more commonly involved in infecting functionally asplenic patients.
A. Page 170 discusses EBV.

This boy has EBV induced mononucleosis, which presents with atypical CD8+ reactive T cells.
A. Page 107 discusses complement disorders.

This question is an error though. This patient presumably has hereditary angioedema.
Hereditary angioedema does not have a rash (urticaria), which is the main differentiating point between angioedema and allergy.
The question writer may not have been aware of this.

If you see angioedema and laryngospasm, always think of C1 esterase inhibitor deficiency.
Choice B was a trick answer choice. C4 levels would be low but choice B is not about that.
D. Page 594 discusses anesthetics.

People with pseudocholinesterase deficiency may not be able to move or breathe on their own for a few hours after
[fast-acting drugs, such as succinylcholine and mivacurium] are administered.

Choice A and E are related to malignant hyperthermia.


Choice B makes no sense because the vagus nerve does not affect the diaphragm, the phrenic nerve does.
B. Page 239 discusses drug interaction types and page 355 discusses cortisol.

As described by UW-
"Cortisol exerts a permissive effect on many hormones to help improve the response to a variety of stressors.
For example, cortisol increases vascular and bronchial smooth muscle reactivity to catecholamines."
E. Page 140 discusses C diphtheriae.

Elongation factor 2 is related to protein synthesis.


C. Page 457 discusses lead poisoning.

Lead Toxicity :
Causes: disruption of heme synthesis via inhibition of ALA (aminolevulinate dehydratase)
Findings:
A: Anemia B: Basophilic stippling C: Colic D: Diarrhea E: Encephalopathy F: Foot drop G: Gum deposits/growth retardation/gout
H: HTN and hyperurecemia

Other answers:

Zinc deficiency:
Causes: low intake, Crohn's disease
Findings: immune dysfunction, impaired wound healing (for the remodeling phase of wound healing, zinc is needed
by metalloproteinases to break down type III collagen), hypogonadism, diarrhea, dermatitis, alopecia, abnormal taste and smell
Excess is rare... but can lead to n/v/d and abdominal pain

Magnesium deficiency:
Causes: congenital, polyuria, malnutrition, laxative abuse
Findings: tetany, premature ventricular contraction, increased QT interval

B12 deficiency:
Causes: vegan diet, pernicious anemia, pancreatic issues, gastric bypass surgery, malabsorption (Crohn, sprue),
Diphyllobothrium lactum/tapeworm infection
Findings: macrocytic, megaloblastic anemia, neuro symptoms, increased homocysteine and MMA levels in serum

Increased Vitamin D:
Causes: granulomatous disease (sarcoidosis)
Findings: bone resorption, too much calcium, loss of appetite, stupor
C.
C. Page 457 discusses PCT.

PCT is an autosomal dominant deficiency of uroporphyrinogen decarboxylase, leading to blistering photosensitivity and hyperpigmentation.
It is the most common porphyria, exacerbated with alcohol consumption and associated with HepC.

PUVA is used to treat Psoriasis and Vitiligo.


Phototherapy with photochemotherapy (PUVA) is a well-known and well-studied modality for the treatment of psoriasis,
which involves systemic or topical administration of chemicals known as psoralens and administration of ultraviolet light
in increasing dosages after requisite time gap. PUVA is also used in the treatment of widespread vitiligo with moderately
good results, though it is being surpassed by ultraviolet B (UVB), which is equally or slightly more efficacious with fewer side effects.
A. Page 525 discusses Bullous pemphigoid.

This is clearly bullous pemphigoid because that is "bullow" the dermis (subepidermal) and fluid-filled.
But these answer choices are bullshit.
Bullous pemphigoid antigen is actually a hemidesmosome, which is what the autoantibodies attack.
Keep it simple.
The other things C-E are cytoskeletal components.
E. Pages 458-459 discuss coagulation and platelet disorders. Page 444 has the coagulation cascade.
A. Page 483 discusses Erb palsy.

Good resource -- https://geekymedics.com/nerve-supply-to-the-upper-limb/


This patient has compression of the C5-C6 which are the upper part, causing ERB Palsy. Upper- Erber.
The upper trunk has roots from C5-C6 as seen in this patient, and it provides the axillary, suprascapular, and musculocutaneous nerves.
Hence, you lose abduction, lateral rotation, and flexion/supination respectively.
C. Page 47 discusses cell trafficking.
There is a problem with the protein that sends things from the RER to the golgie; COP2.
Due to the mutation, the packaged proteins that should bleb off and be sent to the golgi,
instead accumulate in the RER and dilate it.

Choices A and B make no sense, nor is this a lysosomal storage disorder so knock out E and F.
This also is not a disease related to the SER.
Page 662 discusses thiazides.
Hydrochlorothiazide is a thiazide diuretic => thiazide diuretics are associated with hypokalemia.

What other diuretics are associated with hypokalemia? Loop diuretics.

Why?

Inhibition of Na+ reabsorption occurs in both loop diuretics (inhibit NaK2Cl1 cotransporter) and thiazide diuretics (inhibit NaCl cortransporter).
All of this decreased Na+ increases Aldosterone activity.

Aldosterone upregulates expression of the Na+/K+ ATP antiporter (reabsorb Na+ into body, expel K+ into lumen).
This results in hypokalemia in the body, as seen in this patient.

Aldosterone does one other important thing - activation of a H+ channel that expels H+ into the lumen.

So, given that this patient has hypokalemia, you know there is upregulation of Aldosterone. Do you think her pH would be high, or low?
It would be high because inc. Aldosterone => inc. H+ expelled into the lumen => metabolic akalosis.

Now you understand why both loop diuretics and thiazide diuretics can cause what's called "hypokalemic metabolic alkalosis."
B. Page 47 discusses cellular trafficking.

The N-terminus is where you have signal peptides which define the target of the proteins (think of them as postal code).
Typically, proteins are in the cytoplasm after they are made, but since the have tags at n-terminus, they go to different targets like this case ER.
Now, if the protein is retained in the cytoplasm, this means the mutation disabled the tag (no postal code). Hence, it is kept in the cytoplasm.
The mRNA codons are read from 5' to 3' , and they specify the order of amino acids in a protein from N-terminus (methionine) to C-terminus.
The N-terminal signal peptide is recognized by the signal recognition particle (SRP) and results in the targeting of the protein to the
secretory pathway.

While the N-terminus of a protein often contains targeting signals, the C-terminus can contain retention signals for protein sorting.
The most common ER retention signal is the amino acid sequence -KDEL (Lys-Asp-Glu-Leu) or -HDEL (His-Asp-Glu-Leu) at the C-terminus.
This keeps the protein in the endoplasmic reticulum and prevents it from entering the secretory pathway.
C. Page 262 discusses observational studies.
There are two groups, one that had high TV exposure and the other that had lower TV exposure.
They compared the likelihood of TV exposure and estimated the learning disability in these groups.
This is a classic case-control study, where you are selecting by disease outcome (intellectual disability) and
retroactively look at exposure (TV usage).

Case-control studies are assessed with an odds ratio, so in the end you may see a positive relationship and say
something like- "Children with intellectual disabilities were 3 times more likely to have 2 or more hours of TV time
per day than children without intellectual disabilities."

Also, the word "likelihood" is synonymous to "odds" and that also helped.

In the outbreak setting, the term attack rate is often used as a synonym for risk.
It is the risk of getting the disease during a specified period, such as the duration of an outbreak.
A variety of attack rates can be calculated. Overall attack rate is the total number of new cases divided by the total population.

In epidemiology, the attack rate is the biostatistical measure of frequency of morbidity, or speed of spread, in an at risk population.
It is used in hypothetical predictions and during actual outbreaks of disease.
F. Page 587 discusses visual field defects.

For upper quandrantanopias, it is the temporal lobe, while lower


quandrantanopias are the parietal lobe. This image shows you that
the left visual field goes to the right visual cortex, and vice versa.

This patient has a right lower quadrantanopia, hence it must be


the left parietal lobe that is affected.
F. Page 759 discusses lung cancer and page 372 discusses SIADH.
This patient has a small cell carcinoma, which can produce one of many things such as ACTH, SIADH, or
Lambert Eaton. They are describing Kulchitsky cells, which are small dark blue cells.

If we assume from the answer choices that this patient might have SIADH, then we can expect
euvolemic hyponatremia, and you can pick answer F.

No answer fits ACTH or LE related issues.


B. Page 40 discusses DNA repair.
This patient has a newly synthesized DNA which needs repair, specifically for colon cancer.
Lynch syndrome has defective mismatch repair. Page 421 discusses MLH1, MSH2, and PMS2.

UV exposure induced pyrimidine dimers are fixed with nucleotide excision repair (choice C).
This would be defective in Xeroderma pigmentosum.

Toxic alterations are repaired with base excision repair.

BRCA is repaired with a homologous recombinant method.

Ataxia telangiectasia is due to a defect in nonhomologous end joining.


A. Page 302 discusses splitting. Page 317 discusses ASD.
Fixed splitting is iconic for an ASD, where you have high right heart volumes.
The ASD causes a L to R shunt because sending blood into the RA is easier than sending it into the LV.
The ASD causes a diastolic rumble due to increased flow through the TV.
And as in our question, you can also hear a midsystolic pulmonary ejection murmur.
The ASD has your AV bundle, also known more commonly as the bundle of His.
D. PLASMA cells (mature B cells, the ones found in multiple myeloma) secrete antibodies,
but IMMATURE B cells have antibodies that haven't switched classes yet (IgM and IgD).

Since plasma cells do not have an idiotype on their surface, anti-idiotypic antibodies will not work.
A. This is not in FA.
The patient is euthyroid (normal T4/TSH) and sick. Lol.
Cant be D, cuz TSH would be high. Cannot be E, because T3 and T4 would be high.

https://www.ncbi.nlm.nih.gov/books/NBK482219/
The euthyroid sick syndrome, also known as nonthyroidal illness syndrome, refers to changes seen in
patient thyroid function tests administered in the medical intensive care unit during episodes of critical
illness. It is not a true syndrome, and there are significant alterations in the hypothalamic-pituitary-thyroid
axis in about 75% of the hospitalized patients.

Causes of euthyroid sick syndrome vary to include critical illness, pneumonia, starvation, anorexia nervosa, sepsis, stress, history of
trauma, cardiopulmonary bypass, myocardial infarction, malignancies, congestive cardiac failure, hypothermia, inflammatory bowel
disease, cirrhosis, major surgery, renal failure, and diabetic ketoacidosis.
E. Page 649 discusses Nephrotic syndrome.
This patient took penicillamine and developed proteinuria.
Membranous nephropathy can occur after taking NSAIDs, Penicillamine, or gold.
It can also occur in HBV, HCV, and syphillis as well as in SLE. SLE usually has DPG, also called lupus nephritis with "wire lupus."
Membranous nephropathy has a spike and dome appearance due to subepithelial depositis of immune complexes.

Choice A is a nephritic syndrome that occurs in children 2-4 weeks after GAS infection.
IgA nephropathy (Berger disease) is a nephritic syndrome with hematuria, respiratory, and GI infections leading to mesangial IC deposits.
Membranoproliferative is a nephritic syndrome seen in HBV or HCV infection and has subendothelial IC deposits too.
MCD is the only other nephrotic syndrome and that has no deposits, just effacement of foot processes.
D. Page 471 discusses thrombolytics.
This patient has a DVT due to her Afib. Now that she already has an embolus causing a clot, we must remove it.

Clopidogrel is an ADP receptor inhibitor that prevents gp2b/3a on platelets to decrease risk of thrombotic strokes.
Bosentan; -en and -tan, endothelin antagonist. Its used for pulmonary HTN.
Aspirin is also used by afib patients to decrease the risk of thrombotic strokes.
Fasciotomy is a limb-saving procedure when used to treat acute compartment syndrome.
B. Page 384 discusses the retroperitoneal structures.
The ascending and descending colons are the only sad pucker structures of the colon.
B. Page 176 discusses Rabies virus.
This is a bullet shaped -ssRNA virus that forms
cytosplasmic Negri bodies in Purkinje cells of the cerebellum
and hippocampal neurons.

This girl probably got the infection from bats in the cave.
D. Page 612 discusses Personality disorders.
This patient is clearly Schizoid, because hes like an android.
No emotional expression, and he is content being isolated.
E. Page 169 discusses Poxvirus.
This is due to Molluscum contagiosum.
Poxvirus is the largest DS linear DNA virus and it is also unique for making its own envelope and using a
DNA-dependent RNA polymerase to do replication in the cytosol (while others replicate in the nucleus).
The description of umbilicated papules helps make the answer choice.
F. I know where the sphenoethmoid area is and its above the nose
so I cancelled everything down to F. Some people felt D was also
right but F is more correct. Overall, bullshit question.

This video gives a lesson the anatomy of the nasal cavity.


https://www.youtube.com/watch?v=mf7rY1VNy70
E. Page 170 discusses VZV HHV3.
HHV3 causes shingles when it reactivates in the immunosuppressed.
A is an autoimmune condition.
B is also autoimmune and seen in Celiac patients.
C is seen in HSV1.
C. Page 327 discusses cardiomyopathies. Peripartum cardiomyopathy is in FA2019 (pg 305).

Jugular venous distention = Right heart failure

Pulmonary edema = Left heart failure

Four-chamber dilation is the most likely answer. MANY things cause dilated cardiomyopathy.
Peri- or postpartum cardiomyopathy (PPCM) is a rare, life-threatening heart disease of unclear origin and is characterized by
heart failure of sudden onset between the final weeks of pregnancy and 6 months after delivery.

Assymetric septal hypertrophy, myocardial disarray: these are both classic findings in hypertrophic cardiomyopathy (HCM)
Endocardial fibroelastosis: a rare restrictive cardiomyopathy seen in infants/children
Lymphocytic infiltration of the myocardium: seen in viral (autoimmune) myocarditis. A cause of dilated cardiomyopathy, but there
was no mention of a preceding viral illness.
D. Page 677 discusses Cleft lip and palate.

The stem says that neither the woman nor her great-uncle had any other abnormalities,
so they are examples of a "non-syndromic" cleft lip/palate, which accounts for 70% of all
cleft lips/palates. It runs in families but has no distinct inheritance pattern and is considered multifactorial.
E. Page 432 discusses GIT pharmacology.
Tricky bullshit wording. FA teaches that misoprostol helps fix gastric mucosa.
We need to fix the esophageal mucosa, which will repair once the stressor (acid) decreases.
Omeprazole heals gastric and duodenal ulcers more effectively than misoprostol,
whereas misoprostol was more effective in patients with erosions alone.
This patient has GERD. Misoprostol (what I fell for) is for NSAID induced ulcer/erosions.

https://www.nejm.org/doi/full/10.1056/NEJM199803123381105
B. Page 503 discusses injuries of the elbow.
Tennis causes repetitive extension which injures an extensor, and this was the only extensor.
This is also taught by a uworld question. All the muscles there have extensor in the name.
Medial epicondylitis is related to the pronator, flexor carpi radialis, and carpi ulnaris.
B. Page 286 discusses more about Patent Foramen Ovale.
This patient had a "paradoxical" emboli.
E. Page 746 discusses restrictive respiratory diseases.

Usual interstitial pneumonitis is the histological definition of Idiopathic pulmonary fibrosis.


We know that this patient has pulmonary fibrosis because the question states that there is fibrous thickening
of the alveolar septa. This question was just testing that we knew the other names for Pulmonary Fibrosis.
The question was posed with a chronic issue which made it confusing, but there are multiple COPD choices,
and no detail is given for a specific COPD choice so they must be wrong.
B. Page 266 discusses incidence.

So the first screening gave you the prevalence, which was 500/2500, and now we need the incidence of new cases.
This means 200 new events took place in the 2000 remaining students who did not have C trachomatis.
That is 10%.
A. Page 138 discusses E faecalis.
Question asked for gram positive cocci in CHAINS. S. aureus forms clusters, eliminating it.
This leaves Enterococcus faecalis and Group A strep.
E. faecalis is associated with UTIs. "Do U(uti) heart (cardiac) trees (biliary tree)?"
A. Page 57 discusses Hardy Weinberg.

You have to use the hardy weinberg formula (1=p^2+2qp+q^2) and p + q = 1


They basically tell you that q^2=1/900 which makes q=1/30 now you can figure out (p=1-q)
So p=1-(1/30), p=29/30 then to figure out carrier you solve for 2qp, 2(29/30)(1/30)=1/15 if you estimate the 29/30=1.
A. Page 697 discusses AIS.
This patient has no Barr bodies, which means this patient is XO or XY.
Turner syndrome would have a uterus and streak ovaries and not be "normal appearing" but rather be short and flat chested with a broad neck.
Congenital adrenal hyperplasias present much earlier.
Prenatal exposure to excess androgen or estrogen would not cause a lack of a uterus and ovaries, which are the main internal female organs.

Page 678 discusses sexual differentiation.


The medulla of undifferentiated gonads gives you the testes if you have the XY SRY gene.
Testes secrete testosterone from Leydig cells and MIF from sertoli cells.
Testosterone then causes the Wolffian duct to give internal male genitalia, and 5a-Reductase produces DHT for external male genitalia.
MIF stops the Mullerian duct from giving the internal female genitalia.

The cortex of undifferentiated gonads gives you the ovaries if you have XX and no SRY gene.
Ovaries make estradiol, which leads to the development of external female genitalia.
The Mullerian duct gives the internal female genitalia in the absence of MIF.

Problems here can be due to a lack of MIF in a male, leading to a male having both male and female internal genitalia (but male external),
or due to a lack of 5a-Reductase leading to a lack of DHT so the man will lack external male genitalia. Either way, problems here are in men.
D. Page 684 discusses Urethral injury.
This patient has blood at the urethral meatus and probably suffered an posterior membranous urethral injury.

Posterior membranous urethral injuries are related to a pelvic fracture and urine leaks into the retropubic space (Bucks fascia injury).
Would also have blood at the urethral meatus and the prostate would be higher.

Anterior spongy urethral injury is another case.


Blood leaks into the scrotum and urethral meatus. Often related to a perineal saddle injury (bike bar, or fence).

Catheterization is contraindicated in both cases.


D. Page 433 discusses Loperamide.
This drug has no CNS penetration which is why junkies will never abuse it in a question.
This drug is discussed in GIT rather than in Neurology because it does not really help with pain, it slows gut motility.
Hence, it is useful for diarrhea, but like all opioids, it causes constipation.

Page 595 discusses opioids.


Buprenorphine is an "orphan" and has no family to be a "part" of, so it is a partial agonist.
Notably, buprenorphine has low efficacy but high potency so it can even prevent other opioids from binding.
Hence, it can act as an opioid antagonist in the presence of full agonists and cause withdrawal.

Codeine and Morphine are full agonists that junkies would abuse!
Hydrocodone is a synthetic opioid (codeine, morphine, etc are natural).
C. Page 270 discusses the confidence interval.
Remember that the CI= X +/- Z(SE) where SE=(SD/n^.5) and SE decreases as n increases, and vice versa.
In our question, n, the sample size, is decreasing because we will do fewer measurements of the BP.
This means SE will increase, so the CI will increase.

Basically, with fewer measurements, the CI gets wider because you need a larger sample size for more valuable results.
A. This patient has terminal insomnia, which is seen in depression and normal aging.
She has no hypnagogic/hypnopompic symptoms of narcolepsy (pg 605 and 615).
She does not seem to have nocturnal dyspnea (breathless awakening), which is usually due to left heart failure.
She does not have obesity or symptoms of snoring for sleep apnea.
D. Page 764 discusses cromolyn sodium.
This drug prevents mast cell degranulation and helps in acute asthma symptoms.
Histamine H1 blockers would be good for allergy but choice A is a bait.
H2 blockers (-dine meds, dinner for 2) are used for gastric acid. (pg 432).
There are many phosphodesterase inhibitors, and in respiratory we use PDE5 inhibitors for vasodilation; treat pulmonary HTN.
E. Page 721 discusses NRDS.
This premature baby needs oxygen supplementation.
Therapeutic oxygen can cause retinopathy, intraventricular hemorrhage, or bronchopulmonary dysplasia.
The only answer relevant to any of that is E.
Na+ = Low
NH4+ = High (no need for Ammonia buffer).

B. Page 375 discusses DKA. Page 349 discusses insulin.

In diabetic ketoacidosis, there is increased acid in the extracellular space.


Transcellular shift due to decreased insulin leads to more H+ entering the cell in exchange for K+.
This leads to hyperkalemia with depleted intracellular stores of K+.
There is also osmotic diuresis that leads to increased K+ loss in the urine and total body K+ depletion.
The question asks that is most likely to decrease with insulin therapy:
serum potassium concentration will decrease as K+ is now exchanged for H+ inside the cell.

Insulin increases cellular uptake of K+ and amino acids.

Additional UW fun facts regarding Potassium and DKA: use caution giving insulin and IV fluids to dehydrated hyperglycemics
because insulin forces K in cells causing fast decrease of extracellular Potassium, thus give K supplementation even when serum K elevated.
D. Page 81 discusses amino acids.
This patient is most likely at risk of lacking an essential amino acid due to her country.
PVT TIM HaLL is not easy to use.

Phenylalanine, Valine, Tryptophan, Threonine, Isoleucine, Methionine, Histidine, Leucine, Lysine.

Met His Val-entine reminds us that those 3 are glucogenic.


Leucine and Lysine are ketogenic and given to those with PDH deficiency.

The acidic ones are not essential; aspartate and glutamate. (Eliminate choice B).

Choice A - Tyrosine comes from phenylalanine.


C - Our liver can make glycine using glycine synthase.
A. Page 473 discusses Bleomycin.
Side effects include pulmonary fibrosis and hyperpigmentation, as seen.
It does not really cause myelosuppression and it is the only drug that affects G2.
It is used for testicular cancer and Hodgkin lymphoma.

Carboplatin is a platinum agent that is cell cycle independent and helps with testicular, bladder, ovary, GI, and lung cancer.
Adverse effects include Fanconi nephrotoxicity that you can prevent with amifostine and Cl- diuresis, as well as ototoxicity.

Vincristine crisps your nerves and is a M phase microtubule inhibitor for solid tumors, leukemias, and Hodgkin and non-H lymphomas.
G. Page 149 discusses Syphillis.
This is always treated with penicillin, even if they are allergic to penicillin (desensitize first).
C. Page 652 discusses kidney stones.

Remember SIXteine. Cysteine stones have 6 sides; hexagonal.


As stated in the question, the sodium cyanide nitroprusside test will be positive.
Its due to a PCT defect that leads to poor absorption of COLA, Cystine, Ornithine, Lysine, and arginine.

All kidney stones have low pH except the PHOSPHATE stones (ammonium phosphate or calcium phosphate).
Remember PHosPHate has PH in it twice, so high pH.

Calcium oxalate is dumb-bell shaped (lift weights to be strong like an ox- oxalate).
Calcium phosphate is wedge shaped.
Ammonium Magnesium phosphate is coffin shaped.

Urate stones are rhomboid rosettes.


Cystine stones are six sided hexagons.
D.

Familial dysplastic nevus syndrome is inherited AD and characterized by tons of dysplastic nevi with transformation to malignant melanoma.

Acanthosis nigricans is in both areas (sun neck and non sun armpit) but is related to diabetes and obesity.
BCC is only on the upper lip area.
Blue nevi (wtf is this) https://en.wikipedia.org/wiki/Blue_nevus
Pigmented seborrheic keratosis is seen in older people and can be due to GI adenocarcinoma if its multiple like Leser Trelat.
C. Page 714 discusses Tamoxifen.

Tamoxifen-
Selective estrogen receptor modulator (SERM) that acts as an antagonist on
the estrogen receptors of the breast and acts as an partial agonist on estrogen receptors
in bone and uterus. Therefore, ↑ risk of endometrial cancer and ↑ Risk of uterine sarcoma.
Uses: ER or PR positive tumors (breast).

Raloxifene-
Antagonist at breast, uterus; agonist at bone; risk of thromboembolic events (especially
with smoking) but no increased risk of endometrial cancer (vs tamoxifen); used primarily
to treat osteoporosis.
1- Left gastric artery
E. Page 394 discusses the Celiac artery. a] Esophageal branch for distal esophagus.
Left gastric itself goes to the lesser curvature.
This patient has a splenic vein thrombosis.
2) Common hepatic artery
This means a branch of the splenic vein is where the bleeding is coming from. a] Proper hepatic
There are only 2 major branches, and E is one of them. -->left hepatic, right hepatic
-->cystic artery to gallbladder
b] Right gastric to lesser curvature
c] Gastroduodenal
--> Anterior & Posterior superior pancreaticoduodenal
--> Right gastroepiploic to right greater curvature

3) Splenic artery
a] Short gastric to fundus
b] Left gastroepiploic to left greater curvature
C. Page 102 discusses macrophage lymphocyte activation. Page 108 discusses more about IFNg. Page 109 discusses all interferons.

Th1 cells secrete IFNg to activate macrophages.


Granulomas are formed by IFN gamma Maintained by TNF alpha.
Gamma forms alpha maintains. That is why infliximab and etarnacept (TNF alpha inhibitors) will break down the granuloma if used.
D. Page 383 discusses pyloric stenosis.
This patients physical findings are classic for pyloric stenosis.

Duodenal atresia due to failure of recanalization would have a double bubble from the dilated stomach and duodenum.
C. Page 264 explains evaluation of diagnostic tests.
Understand that when everyone around you has a disease, then a positive result is expected.
And if no one has the disease, a negative test is expected.

The FNr and FPr are related to sensitivity and specificity respectively.
Sensitivity is your TPrate, which is 1-FNr.
Specificity is your TNrate, which is 1-FPr.
Sensitivity and specificity are inherent to the study not the disease or its prevalence/incidence.
D. Page 88 discusses lysosomal storage diseases.
This patient has a cherry red macula, which is only seen in Tay Sachs and Niemann Pick.
Hyperaccusis is a feature in Tay Sachs patients.
Hepatosplenomegaly is seen in Niemann pick and was not seen in this patient, so TS is likely.

Neimann Pick would accumulate sphingomyelin because you have no sphingomyelinase to convert it to ceramide.

Glucocerebroside is made by both HexA (tay sachs) and alpha-galactosidase (fabry).


This is what accumulates in Gaucher disease because they cant further convert it to ceramide using glucocerebrosidase.

Ceramide does not accumulate, it is the final product.


C.

Without the ovaries there will be no feedback inhibition so FSH LH will continue being secreted.
The stress of no feedback will cause more FSH LH secretion (gonadotrophs).
D. Page 119 discusses transplant rejection.
This patient has a hyperacute reaction which is a hypersensitivity (type 2).
The answer must be D because only IgG and IgM fix complement. IgA is not related to a hypersensitivity per se.
CD4 and CD8 are BOTH part of acute and chronic rejections.
E.

The patient had seizures, which occurs due to hyponatremia or hypocalcemia.


This is why so many antiepileptics block Na or Ca channels.
This question is more about the water intoxication in addition to the diarrhea.

Water Intoxication in infants -- https://www.ncbi.nlm.nih.gov/pubmed/1877579


B. Page 364 discusses thyroid.
I think the concept they’re testing is the increased TBG levels in pregnancy, and not just hyperthyroidism in general.

When screening for hypo/hyperthyroidism, TSH levels are ALWAYS preferentially checked because they are more sensitive to
minute differences in T3/T4. Often times TSH levels can demonstrate a change even when T3/T4 levels are in the subclinical range.
The only exception to this would be in pregnancy (and I guess maybe liver failure? I doubt they would ask this though).
High estrogen levels prevents the liver from breaking down TBG, leading to increased TBG levels in the serum.
This binds to free T4, decreasing the amount of available free T4. As a compensatory mechanism, TSH levels are transiently increased
and the RATE of T4 production is increased to replenish baseline free T4 levels. However the TOTAL amount of T4 is increased but
free T4 should be normal.

The question is asking how to confirm hyperthyroidism in a pregnant woman --> you need to check FREE T4 levels
(because they should be normal due to compensatory response). You cannot check TSH (usually elevated in
pregnancy to compensate for increased TBG), and you cannot check total T4 levels (will be increased).
D.

The question is asking you how much water must the woman take in to maintain the same osmolality.
This woman takes in 450 mOsm of solute per day.

For her to keep the same osmolality she must excrete 450 mOsm per day.
The only way for her to excrete the solute is via the kidneys.
The only way for her kidneys to excrete 450 mOsm is if they excrete 1 liter of water also.
This is depicted by the 450/mOsm/day/kg where kg and L are pretty much interchangabel.
This is the max concentration that her kidneys can produce.
(Her kidneys are not “powerful enough” to make her urine any more concentrated than that.)

This woman is also losing another liter of water to feces, sweating and respiration. This is the “insensible water loss”.
That means if she losing 1 liter of water to sweat, respiration and feces per day plus 1 liter of water to urine
(because she needs to dissolve her solute in something!), for her blood to stay the same osmolality,
she must replace the water she lost thus must, at minimum, drink 2 liters of water per day.
B. Page 678 discusses sexual differentiation.

Male internal genitalia -> Intact SRY , testes, and testosterone.

No female internal genitalia -> Presence of MIF (antimullerian hormone) and intact Sertoli cell function.

Female external genitalia in XY -> No androgen present, which is required for male external genitalia formation.
D. Page 721 discusses alveolar cells.
Type 1 pneumocytes only have ONE job, diffusion.
Type 2 have TWO jobs, diffusion and regeneration. Type 2 have lamellar bodies that make surfactant.
Type 2 themselves serve as precursors to more type 1 and 2.
D. Page 312 discusses baro and chemoreceptors.

This patient was running and has a low BP so you know sympathetics will be high and parasympathetics will be low.
This leaves you with A or D. The carotid baroreceptor is a pressure sensor. Since he is hypotensive, this decreases stretch.
Carotid impulses are parasympathetic, and we just said parasympathetics will be low.
Recall that a carotid massage would slow the heart and cause parasympathetics to be in action, decreasing the AV node refractory period.
His pulse is very high so carotid must not be sending impulses.
B. Page 203 discusses ethambutol.

Ethambutol = EYEthambutol, optic neuropathy.

Great mnemonic for remembering that EYEthambutol is the component that causes visual problems in RIPE therapy for TB.
D. Page 262 discusses Clincal Trials.
This is postmarketing surveillance where the drug is now open to the market and we are checking "can it stay?"

Phase 1 has 10^1 healthy people or patients with disease to see if its safe and check toxicity, kinetics, and dynamics.
Phase 2 has 10^2 patients with disease to see if it works and how it works (efficacy, dosing, adverse effects).
Phase 3 has 10^3 patients that allow you to compare the new drug with the current standard.
D. Page 558 discusses strokes.
"Dont PICA (pick a) horse that cant swallow."
This is lateral medullary wallenberg syndrome with nucleus ambiguus symptoms.

AICA - "Facial droop means AICA's pooped (broken)."


This would be lateral pontine syndrome with facial nucleus symptoms.
Basilar artery stroke leads to locked in syndrome with loss of horizontal eye movement.
PCA supplies many portions but does not really have a relevant stroke syndrome.
Superior cerebellar is also not a real issue.
E.

This question is garbage. The writer must be biased against alcohol.


One weekend of heavy drinking will do minimal damage.
Cellular swelling, specially being "acute" is the best answer here.
Cellular swelling indicates alcoholic hepatitis.

However, truth is the order of damage is as below.


As per FA 1) fatty infiltration 2) cellular ballooning 3) eventual necrosis.
But this would not necessarily occur in one weekend.
The question should have been "weekends."
A.

Stretch or dilation of the cervix and vagina are strong stimuli for oxytocin secretion,
mediated by neural pathways called the Ferguson reflex.

Article: https://www.sciencedirect.com/topics/neuroscience/ferguson-reflex
A. Page 404 discusses GI secretory cells.

Gastrin, intrinsic factor, and pepsin are secreted by the stomach.


VIP is synthesized in neurons, so CCK (from I cells of duodenum) would be most directly affected by a duodenectomy.

The duodenum secretes CCK from I cells, secretin from S cells, and GIP from K cells.
CCK increases pancreatic secretions and relaxes the sphincter of Oddi, which is what the 2nd part of the duodenum is related to.
A. Page 317 discusses PDA.
Blood flows from the descending aorta back into the pulmonary arteries.
The "steal" from the aorta during diastole requires increased cardiac output to compensate.
Extremely premature infants have limited ability to increase stroke volume and thus use increased heart rate to increase cardiac output.

B is nonsense. C will not be affected as blood is neither entering or leaving the RV. D is mainly unaffected.
E is a trick but systemic arterial O2 will actually be normal in the upper extremities; differential cyanosis.
AMBOSS= A 50-year-old man undergoes parathyroidectomy for treatment-resistant hyperparathyroidism. The procedure is complicated by brisk bleeding from an artery that travels
along the external branch of the superior laryngeal nerve. To stop the bleeding, the artery is ligated at its origin. Which of the following is most likely the origin of the artery that was
injured in this patient? External Carotid Artery (39%)

The superior thyroid artery travels along the superior laryngeal nerve. The superior thyroid artery is a direct branch of the external carotid artery. After the carotid artery bifurcates in
the carotid triangle, the superior thyroid artery arises as the first branch of the external carotid artery and supplies the cranial half of the thyroid gland before forming an anastomotic
network with the inferior thyroid artery. The artery travels with the external branch of the superior laryngeal nerve and pierces the thyroid gland at the superior pole.
Therefore, bleeding from the superior thyroid artery would be controlled by ligating it at its origin from the external carotid artery.

27% chose the Thyrocervical Trunk. The thyrocervical trunk originates from the subclavian artery and gives rise to the inferior thyroid artery. The inferior thyroid artery pierces the
lower pole of the thyroid gland and is closely associated with the recurrent laryngeal nerve. This patient has an injury of a vessel traveling with the superior laryngeal nerve rather
than with the RLN.

The subclavian artery gives rise to the inferior thyroid artery via the thyrocervical trunk. The inferior thyroid artery pierces the lower pole of the thyroid gland and is closely
associated with the recurrent laryngeal nerve. This patient has an injury of a vessel traveling with the superior laryngeal nerve rather than with the RLN.

Superior Thyroid artery from External carotid; superior laryngeal nerve.


Inferior Thyroid artery from Thyrocervical trunk; recurrent laryngeal nerve.
A. Page 349 discusses GLUT5
This patient has normal hormones. GLUT5 is for sending fructose to spermatocytes and GIT.
Fructose makes up 99% of the reducing sugar present in semen.

Selenium is a content of semen, but levels vary so much that it serves no prognostic clincal relevance to infertility,
per this article: https://www.ncbi.nlm.nih.gov/pubmed/3235210.

Zinc deficiency (in serum), per FA, has been implicated in delayed wound healing, suppressed immunity, male
hypogonadism, decreased adult hiar (axillary, facial, pubic), dysgeusia, anosmia, acromdermatitis enteropathica
(defect in intestinal zinc absorption), and may predispose to alcoholic cirrhosis. FA didn't specify the levels in the
semen, but per the below article, seminal zinc content was found to have no correlation to sperm/semen activity/level/etc.
I don't know if we're supposed to know that, but now I do, lol. https://www.ncbi.nlm.nih.gov/pubmed/3570537.
A.

Craniopharyngioma with Calcifications and Cholesterol Crystals (motor-oil fluid). Remnant of Rathke's pouch.
Most common supratentorial childhood tumor.
Not to be confused with pituitary adenoma.
E.

Patient has positive urine bilirubin test indicating that he has water soluble bilirubin accumulating in his body that has already been
metabolized by UDP-glucuronosyltransferase. In order for water soluble bilirubin to be accumulating in his body there must be defective
excretion of such bilirubin. The only option available that matches such is "obstruction of the bile duct".

Why hemolysis is wrong:

There should almost never be straight up bilirubin in the urine. In hemolysis, the excess bilirubin is excreted in the bile.
After bacterial conversion and reuptake, some will be excreted in the urine as urobilin. However, in obstructive disorders,
the conjugated bilirubin will never have the opportunity to undergo bacterial conversion to sterco/urobilin. In this way, the
conjugated bilirubin has no other way to be excreted other than directly in the urine.
A. Not in FA.
Cysteine-cysteine chemokine receptor 5 (CCR5) is a protein found on the surface of CD4 cells.
Most people never check what CCR5 stands for.
B.

Fats are ketogenic (except odd chain FA), so they produce ketones for energy production (Acetyl-CoA) rather than glucose. If the question asked
what the primary source of energy production was, it would still be glycogen (and not ketones), because this is within 24 hours. However after 24
hours the answer could be ketone bodies. Regardless, the question specifically said the pt had a serum glucose of 100, indicating that we are
looking for something that provides a substrate for gluconeogenesis.

During periods of starvation, substrates for gluconeogenesis come from two sources: (1) breakdown of existing muscle, or (2) via odd-chain FA
through propionyl-CoA. (*Valine also feeds into propionyl CoA, but is not involved during starvation --> see below)

(1) The alanine-pyruvate cycle provides this (glutamine in muscle + pyruvate --> alanine --> goes to liver --> transamination to alpha-ketoglutorate
--> pyruvate is separated from glutamine --> glutamine goes to urea cycle, pyruvate goes on to gluconeogenesis). Lactate can also be used (this
could have been a right answer if it were listed).

(2) Odd chain FAs are also glucogenic, but stearic acid (provided in the answer choice) isn’t odd chain, so it is only ketogenic and can be ruled
out.

Although valine (and other branched a.a.) feed into Propionyl-CoA, they are not used in starvation because starvation strictly relies on hepatic
gluconeogenesis. These a.a. are not metabolized in the liver because the liver lacks branched-chain a.a. transferase enzyme. In First Aid,
Biochem section, under Fasting/Starvation, in both the “fasting state” (which is within the time frame of this question), or the “starvation state,”
both utilize hepatic gluconeogenesis. My assumption is that valine is used during regular metabolism, and not during periods of starvation.
A.

Renal artery stenosis is going to decrease blood flow to the kidney.


JG cells sense the decrease in perfusion pressure and secrete renin.

Renin is produced by the JG cells, JG cells are in the cortex (they are modified smooth muscle of the afferent arteriole).

I think this one is literally just asking what part of the kidney will be the most poorly perfused. That part would have the most renin.
Also, the medulla doesn’t have JG cells so I guess that’s another reason why it couldn’t have the most renin.
D.

Pulmonary fibrosis increases elastic recoiland widens airway secondary to increased outward force (radial traction) by fibrotic tissue
thus decreasing airflow resistance thus supernormal expiratory flow rates (higher than normal following correction for lung volume).
B.
F
C
C.

Loop diuretics are first line for acute congestive heart failure. That should help you remember that they
are the most potent diuretics, so they're often used in the acute treatment of edema.

Chronic renal failure = High potassium in blood and Furosamide is the strongest diuretic on
that list that also depletes Potassium.
A
A

APocrine = your armpits smell like an APE


ceRUMen = there’s no ROOM in your ears since they’re full of wax
EC-CRYne = when you ECercise, your pores are CRYing
SEBaceous = SEBum is SEEPing out of your pores
Holocrine = Acne
Exocrine = breast

APOCRINE vs. ECCRINE

Your skin has two types of sweat glands: eccrine and apocrine. Eccrine glands occur over most of your body and open directly
onto the surface of your skin. Apocrine glands open into the hair follicle, leading to the surface of the skin. Apocrine glands
develop in areas abundant in hair follicles, such as on your scalp, armpits and groin.
B.
Hemostasis (blood clotting): Within the first few minutes of injury,
platelets in the blood begin to stick to the injured site.
This activates the platelets, causing a few things to happen.
They change into an amorphous shape, more suitable for clotting, and they release chemical signals to promote clotting.
This results in the activation of fibrin, which forms a mesh and acts as "glue" to bind platelets to each other.
This makes a clot that serves to plug the break in the blood vessel, slowing/preventing further bleeding.[5][6]

Inflammation: During this phase, damaged and dead cells are cleared out, along with bacteria and other pathogens or debris.
This happens through the process of phagocytosis, where white blood cells "eat" debris by engulfing it.
Platelet-derived growth factors are released into the wound that cause the migration and division of cells during the proliferative phase.

Proliferation (growth of new tissue): In this phase, angiogenesis, collagen deposition, granulation tissue formation, epithelialization, and wound
contraction occur.
In angiogenesis, vascular endothelial cells form new blood vessels.
In fibroplasia and granulation tissue formation, fibroblasts grow and form a new, provisional extracellular matrix (ECM) by excreting collagen
and fibronectin.
Concurrently, re-epithelialization of the epidermis occurs, in which epithelial cells proliferate and 'crawl' atop the wound bed, providing cover
for the new tissue.
In wound contraction, myofibroblasts decrease the size of the wound by gripping the wound edges and contracting using a mechanism that
resembles that in smooth muscle cells.
When the cells' roles are close to complete, unneeded cells undergo apoptosis.

Maturation (remodeling): During maturation and remodeling,


collagen is realigned along tension lines, and cells that are no longer needed are removed by programmed cell death, or apoptosis.

Approximate times of the different phases of wound healing,[10] with faded intervals marking substantial variation, depending mainly on wound
size and healing conditions, but image does not include major impairments that cause chronic wounds.
E . The cervix is the only structure that would result in bilateral blockade.

Invasive cervical carcinoma is associated with hydronephrosis and renal failure due to
cancer spreading through uterine wall and into the bladder.
D.

Elevations in body temperature occur when concentrations of prostaglandin E(2) (PGE(2)) increase within certain areas of the brain.These
elevations alter the firing rate of neurons that control thermoregulation in the hypothalamus. It is now clear that most antipyretics work by
inhibiting the enzyme cyclooxygenase and reducing the levels of PGE(2) within the hypothalamus.

You might also like